Skip to content

Commit

Permalink
fix small typos
Browse files Browse the repository at this point in the history
  • Loading branch information
mbertucci47 authored and aisejohan committed Jun 18, 2024
1 parent 9daea86 commit 0930990
Show file tree
Hide file tree
Showing 88 changed files with 285 additions and 285 deletions.
2 changes: 1 addition & 1 deletion adequate.tex
Original file line number Diff line number Diff line change
Expand Up @@ -2620,7 +2620,7 @@ \section{Pure extensions}
\medskip\noindent
Proof of (2). Because $P_0$ is pure projective
we can find a map $P_0 \to N_0$ lifting the map $P_0 \to M \to N$.
We obtain an induced map $P_1 \to F_0 \to N_0$ wich ends up in $K_0$.
We obtain an induced map $P_1 \to F_0 \to N_0$ which ends up in $K_0$.
Since $P_1$ is pure projective we may lift this
to a map $P_1 \to N_1$. This in turn induces a map
$P_2 \to P_1 \to N_1$ which maps to zero into
Expand Down
12 changes: 6 additions & 6 deletions algebra.tex
Original file line number Diff line number Diff line change
Expand Up @@ -3922,7 +3922,7 @@ \section{Open and closed subsets of spectra}
$R$ annihilated by a positive power of $e'$. Since $e'$ is
idempotent this is the ideal of elements annihilated by $e'$
which is the ideal $I = (e)$ as $e + e' = 1$ is a pair
of orthognal idempotents. This proves (2).
of orthogonal idempotents. This proves (2).
\end{proof}


Expand Down Expand Up @@ -10297,7 +10297,7 @@ \section{Separable extensions, continued}
is a transcendence base of $K/k$. Thus $L$ is the fraction field
of the polynomial ring over $k$ in
$x_1, \ldots, x_{i - 1}, x_{i + 1}, \ldots, x_{n + 1}$.
By Gaus' Lemma we conclude that
By Gauss' Lemma we conclude that
$$
P(T) =
F(x_1, \ldots, x_{i - 1}, T, x_{i + 1}, \ldots, x_{n + 1}) \in L[T]
Expand Down Expand Up @@ -13135,7 +13135,7 @@ \section{K-groups}
By Lemma \ref{lemma-K0-local} the module $P$ is finite free. So
$P \cong R^{\oplus n}$ for some $n \geq 0$. Then $\text{rank}_R(P) = n$ and
$\text{length}_R(R^{\oplus n}) = n \text{length}_R(R)$
by additivity of lenghts (Lemma \ref{lemma-length-additive}).
by additivity of lengths (Lemma \ref{lemma-length-additive}).
Thus the result holds.
\end{proof}

Expand Down Expand Up @@ -15580,7 +15580,7 @@ \section{Relative assassin}
\item $B_{fin}$: for some finite $R$-module $M$ we have
$\mathfrak q \in \text{Ass}_S(N \otimes_R M)$.
\end{enumerate}
Let us determine some of the relations between theses sets.
Let us determine some of the relations between these sets.

\begin{lemma}
\label{lemma-compare-relative-assassins}
Expand Down Expand Up @@ -33688,7 +33688,7 @@ \section{Differentials}
Since $\varphi$ is surjective, the terms
$s_j'[a_j', b_j']$ and $s_k'[f_k', g_k']$ are in the image
of elements in the lower right corner. Thus, modifying
$\eta$ and $\theta$ by substracting the images of these
$\eta$ and $\theta$ by subtracting the images of these
elements, we may assume $\theta = \sum s_l'[r_l']$.
In other words, we see $\sum \varphi(s_i)[\varphi(a_i)]$ is of the form
$\sum s'_l [\beta(r'_l)]$.
Expand Down Expand Up @@ -43445,7 +43445,7 @@ \section{Serre's criterion for normality}
Lemma \ref{lemma-characterize-zero-local}.
This implies that $R_{\mathfrak p}$ is a subring of a product of
localizations of $R$ at $\mathfrak p \supset \mathfrak q$ with
$t \not \in \mathfrak q$. Since theses primes have smaller height
$t \not \in \mathfrak q$. Since these primes have smaller height
by induction on the height we conclude that $R$ is reduced.
\end{proof}

Expand Down
16 changes: 8 additions & 8 deletions algebraization.tex
Original file line number Diff line number Diff line change
Expand Up @@ -1716,7 +1716,7 @@ \section{The theorem on formal functions}
see Remark \ref{remark-compare-with-completion}.
Derived completion commutes with passing to open subschemes
(Remark \ref{remark-localization-and-completion}).
Formation of $R\lim$ commutes with passsing to open subschemes.
Formation of $R\lim$ commutes with passing to open subschemes.
It follows that to check our map is an isomorphism, we may work locally.
Thus we may assume $X = U = \Spec(A)$.
Say $I = (f_1, \ldots, f_r)$. Let
Expand Down Expand Up @@ -3381,7 +3381,7 @@ \section{Algebraization of local cohomology, III}
We want to show that $\xi$ maps to zero in $H^{s + 1}_T(I^mM)$.
Namely, this will show that $\{H^s_T(M/I^nM)\}_{n \geq 0}$
is Mittag-Leffler exactly as in the proof of Lemma \ref{lemma-final-bootstrap}.
Picture to help vizualize the argument:
Picture to help visualize the argument:
$$
\xymatrix{
&
Expand Down Expand Up @@ -3660,7 +3660,7 @@ \section{Algebraization of formal sections, II}
\label{section-algebraization-sections-coherent}

\noindent
It is a bit difficult to succintly state all possible
It is a bit difficult to succinctly state all possible
consequences of the results in
Sections \ref{section-algebraization-sections-general} and
\ref{section-bootstrap}
Expand Down Expand Up @@ -3711,7 +3711,7 @@ \section{Algebraization of formal sections, II}
corresponds to a module map $M' \to M$, see
Cohomology of Schemes, Lemma \ref{coherent-lemma-homs-over-open}.
After replacing $M'$ by the image of $M' \to M$
and seting $M'' = M/M'$ we see that our short exact sequence
and setting $M'' = M/M'$ we see that our short exact sequence
corresponds to the restriction of the short exact sequence of
coherent modules associated to the short exact sequence
$0 \to M' \to M \to M'' \to 0$ of $A$-modules.
Expand All @@ -3733,7 +3733,7 @@ \section{Algebraization of formal sections, II}
Choose a presentation $A^{\oplus m} \to A^{\oplus n} \to J \to 0$.
Denote $g_1, \ldots, g_n \in J$ the images of the basis vectors
of $A^{\oplus n}$, so that $J = (g_1, \ldots, g_n)$. Let
$A^{\oplus m} \to A^{\oplus n}$ be given by the matric $(a_{ji})$
$A^{\oplus m} \to A^{\oplus n}$ be given by the matrix $(a_{ji})$
so that $\sum a_{ji} g_i = 0$, $j = 1, \ldots, m$.
Since $M \to M''$ is surjective, for each $i$ we can choose $m_i \in M$
mapping to $\varphi(g_i) \in M''$. Then the element $g_i \hat s - m_i$
Expand Down Expand Up @@ -3762,7 +3762,7 @@ \section{Algebraization of formal sections, II}
\end{proof}

\noindent
The following lemma will be superceded by
The following lemma will be superseded by
Proposition \ref{proposition-application-H0}.

\begin{lemma}
Expand Down Expand Up @@ -4051,7 +4051,7 @@ \section{Algebraization of formal sections, II}
Since $f$ is a nonzerodivisor on both $N$ and $M$ we conclude
that $M_g \to N_g$ induces an isomorphism on $f$-adic completions
which in turn implies $M_g \to N_g$ is an isomorphism in an open
neightbourhood of $V(f) \cap D(g)$.
neighbourhood of $V(f) \cap D(g)$.
Since $g \in \mathfrak a$ was arbitrary, we conclude that $M$ and $N$
determine isomorphic coherent modules over an open $V$
as in the statement of the lemma. This finishes the proof.
Expand Down Expand Up @@ -7447,7 +7447,7 @@ \section{Algebraization of coherent formal modules, V}
\delta^Y_Z(y) - (d - 1)
\end{aligned}
\end{equation}
This inequality will allow us to check the remaning conditions.
This inequality will allow us to check the remaining conditions.

\medskip\noindent
Conditions (b) and (d) of
Expand Down
12 changes: 6 additions & 6 deletions artin.tex
Original file line number Diff line number Diff line change
Expand Up @@ -48,7 +48,7 @@ \section{Introduction}
associated to $x_0$ over $k$. We introduce the Rim-Schlessinger condition (RS)
for $\mathcal{X}$ and show it guarantees that
$\mathcal{F}_{\mathcal{X}, k, x_0}$ is a deformation category, i.e.,
$\mathcal{F}_{\mathcal{X}, k, x_0}$ satisies (RS) itself.
$\mathcal{F}_{\mathcal{X}, k, x_0}$ satisfies (RS) itself.
We discuss how $\mathcal{F}_{\mathcal{X}, k, x_0}$
changes if one replaces $k$ by a finite extension
and we discuss tangent spaces.
Expand Down Expand Up @@ -3348,7 +3348,7 @@ \section{Infinitesimal deformations}
\text{Mod}_A \longrightarrow \textit{Sets},\quad
M \longrightarrow \text{Lift}(x, A[M])
$$
of isomorphism classes of infintesimal deformations of $x$ to
of isomorphism classes of infinitesimal deformations of $x$ to
$\Spec(A[M])$. We apply Formal Deformation Theory, Lemma
\ref{formal-defos-lemma-linear-functor}
to $\text{Inf}_x$ and $T_x$. This lemma is applicable, since
Expand Down Expand Up @@ -5339,7 +5339,7 @@ \section{Artin's theorem on contractions}
\ref{spaces-cohomology-lemma-proper-over-affine-cohomology-finite} and
\ref{spaces-cohomology-lemma-flat-base-change-cohomology}). Denote
$J'' = \Ker(A \to A')$\footnote{Contrary to what the reader
may expect, the ideals $J$ and $J''$ won't agreee in general.}.
may expect, the ideals $J$ and $J''$ won't agree in general.}.
We have $J_i = J''A[1/f_i]$ as follows
from base change to the spectrum of $A[1/f_i]$.
Observe that we have a commutative diagram
Expand Down Expand Up @@ -5803,7 +5803,7 @@ \section{Artin's theorem on contractions}
Proof of surjectivity: algebra, more ring maps.
Denote $Z_n \subset V$ the $n$th infinitesimal
neighbourhood of $Z$ and denote $Z_{\mu, n} \subset V_\mu$
the $n$th infinitesimal neighbourhoof of $Z_\mu$.
the $n$th infinitesimal neighbourhood of $Z_\mu$.
By the theorem on formal functions
(Cohomology of Spaces, Theorem
\ref{spaces-cohomology-theorem-formal-functions})
Expand Down Expand Up @@ -6156,7 +6156,7 @@ \section{Artin's theorem on contractions}
$W' \to W$ such that the image of $v$ in $W_{red}$ is in the
image of $W'_{red} \to W_{red}$. Then $V_{/Z} \times_{g, W} W' \to V_{/Z}$
is an adic \'etale morphism of formal algebraic spaces over $S$
and $V_{/Z} \times_{g, W} W'$ is an affine fromal algebraic space.
and $V_{/Z} \times_{g, W} W'$ is an affine formal algebraic space.
By Algebraization of Formal Spaces,
Lemma \ref{restricted-lemma-algebraize-rig-etale-affine}
there exists an \'etale morphism $\varphi : V' \to V$ of affine schemes
Expand Down Expand Up @@ -6190,7 +6190,7 @@ \section{Artin's theorem on contractions}
$$
is smooth at $\mathfrak m$ for all $n$. By the discussion above
we may and do assume that $B_1 \to A_1$ is a smooth ring map.
Denote $\mathfrak m_1 \subset A_1$ the maximal ideal corresponing
Denote $\mathfrak m_1 \subset A_1$ the maximal ideal corresponding
to $\mathfrak m$. Since smoothness implies flatness, we see that:
for all $n \geq 1$ the map
$$
Expand Down
2 changes: 1 addition & 1 deletion categories.tex
Original file line number Diff line number Diff line change
Expand Up @@ -3001,7 +3001,7 @@ \section{Essentially constant systems}
\end{lemma}

\begin{proof}
Omitted. This is a good excercise in the definitions.
Omitted. This is a good exercise in the definitions.
\end{proof}

\begin{remark}
Expand Down
22 changes: 11 additions & 11 deletions chow.tex
Original file line number Diff line number Diff line change
Expand Up @@ -273,7 +273,7 @@ \section{Periodic complexes and Herbrand quotients}
0 \to (M, \Im(\varphi), \varphi, 0) \to
(M, N, \varphi, \psi) \to (0, N/\Im(\varphi), 0, 0) \to 0
$$
The intial remark combined with the additivity of
The initial remark combined with the additivity of
Lemma \ref{lemma-additivity-periodic-length}
reduces us to the cases (a) $M = 0$ and (b) $\varphi$ is surjective.
We leave those cases to the reader.
Expand Down Expand Up @@ -457,7 +457,7 @@ \section{Calculation of some multiplicities}
a_2 = a_1 + b_{n - 2} - b_1,\quad
a_3 = a_2 + b_{n - 3} - b_2,\quad \ldots
$$
It is straighforward to see that this implies $a_i = a_{n - i}$ as desired.
It is straightforward to see that this implies $a_i = a_{n - i}$ as desired.
\end{proof}

\begin{lemma}
Expand Down Expand Up @@ -4004,7 +4004,7 @@ \section{Chow groups and K-groups}
$$
\text{length}_A(A/I) - \text{length}_A(A/fI)
$$
Using the distance fuction of
Using the distance function of
Algebra, Definition \ref{algebra-definition-distance}
we can rewrite this as
$$
Expand Down Expand Up @@ -6190,7 +6190,7 @@ \section{Bivariant intersection theory}
c \longmapsto res(c)
$$
obtained by viewing a scheme $Y''$ locally of finite type over $Y'$
as a scheme locally of finite type over $Y$ and settting
as a scheme locally of finite type over $Y$ and setting
$res(c) \cap \alpha'' = c \cap \alpha''$ for any $\alpha'' \in \CH_k(Y'')$.
This restriction operation is compatible with compositions in an
obvious manner.
Expand Down Expand Up @@ -6568,7 +6568,7 @@ \section{Lemmas on bivariant classes}
(j'')^*(c \cap g^*\alpha') =
c \cap (i'')^*g^*\alpha'
$$
by our assuptions on $c$; note that the modified version of (3)
by our assumptions on $c$; note that the modified version of (3)
assumed in the statement of the lemma applies to $i''$
and its base change $j''$. We similarly know that
$$
Expand Down Expand Up @@ -9510,7 +9510,7 @@ \section{Chern classes and the derived category}
(Lemma \ref{lemma-splitting-principle}) we may assume each
$\mathcal{E}^i$ has a filtration whose successive
quotients $\mathcal{L}_{i, j}$ are invertible modules.
Settting $x_{i, j} = c_1(\mathcal{L}_{i, j})$ we see that
Setting $x_{i, j} = c_1(\mathcal{L}_{i, j})$ we see that
$$
c(E) =
\prod\nolimits_{i\text{ even}} (1 + x_{i, j})
Expand Down Expand Up @@ -9624,7 +9624,7 @@ \section{Chern classes and the derived category}
\begin{proof}
After choosing an envelope $f : Y \to X$ such that $Lf^*E$ and $Lf^*F$
can be represented by locally bounded complexes of finite locally
free $\mathcal{O}_X$-modules this follows by a compuation from the
free $\mathcal{O}_X$-modules this follows by a computation from the
corresponding result for vector bundles in
Lemmas \ref{lemma-chern-classes-tensor-product} and
\ref{lemma-chern-character-multiplicative}.
Expand Down Expand Up @@ -11222,7 +11222,7 @@ \section{Two technical lemmas}
by Lemma \ref{lemma-base-change-loc-chern}. The fourth
equality, in which $i'_\infty : W'_\infty \to W'$ is the
inclusion morphism, follows from the fact that $c_p(W' \to W, Q)$
is a bivariant class. For the fith equality, observe that
is a bivariant class. For the fifth equality, observe that
$c_p(W' \to W, Q)$ and $c_p(Z' \to X, F)$
restrict to the same bivariant class in
$A^p((b')^{-1} \to b^{-1}(\mathbf{A}^1_X))$ by
Expand Down Expand Up @@ -15503,7 +15503,7 @@ \section{Change of base scheme}
chow groups $\CH_k(X_i)$ as well as maps
$\colim Z_k(X_i) \to Z_k(X')$ and $\colim \CH_i(X_i) \to \CH_k(X')$.
We may replace $S$ by a quasi-compact open through which $X \to S$
factors, hence we may and do assume all the schemes occuring in
factors, hence we may and do assume all the schemes occurring in
this proof are Noetherian (and hence quasi-compact and quasi-separated).

\medskip\noindent
Expand All @@ -15512,7 +15512,7 @@ \section{Change of base scheme}
be an integral closed subscheme of $\delta'$-dimension $k$. By
Limits, Lemma \ref{limits-lemma-descend-finite-presentation}
we can find an $i$ and a morphism $Z_i \to X_i$ of finite presentation
whose base change is $Z'$. Afer increasing $i$ we may assume $Z_i$
whose base change is $Z'$. After increasing $i$ we may assume $Z_i$
is a closed subscheme of $X_i$, see
Limits, Lemma \ref{limits-lemma-descend-closed-immersion-finite-presentation}.
Then $Z' \to X_i$ factors through $Z_i$ and we may replace $Z_i$
Expand Down Expand Up @@ -15556,7 +15556,7 @@ \section{Change of base scheme}
Arguing as above we can find an $i$ and integral closed subschemes
$W_{i, l} \subset X_i$ of $\delta_i$-dimension $k + 1$
whose base change is $W'_l$.
After increasin $i$ we may assume we have rational functions
After increasing $i$ we may assume we have rational functions
$f_{i, l}$ on $W_{i, l}$. Namely, we may think of $f'_l$ as a
section of the structure sheaf over a nonempty open $U'_l \subset W'_l$,
we can descend these opens by Limits, Lemma \ref{limits-lemma-descend-opens}
Expand Down
10 changes: 5 additions & 5 deletions cohomology.tex
Original file line number Diff line number Diff line change
Expand Up @@ -5073,7 +5073,7 @@ \section{{\v C}ech cohomology of complexes}
}
$$
where the left horizontal arrows are (\ref{equation-global-sections-to-cech}).
Since in this case the horizonal arrows are isomorphisms in the derived
Since in this case the horizontal arrows are isomorphisms in the derived
category (see proof of Lemma \ref{lemma-cech-complex-complex}) it
suffices to show that the left square commutes. This is true because
the map $\gamma$ uses the sign $1$ on the summands
Expand Down Expand Up @@ -7473,7 +7473,7 @@ \section{Cup product}
\text{Tot}(\mathcal{A}^\bullet \otimes \mathcal{B}^\bullet)$.
Finally, the commutativity of the remaining square
is true on the level of complexes and may be viewed as the
definiton of the naive cup product (by the adjointness
definition of the naive cup product (by the adjointness
of $f^*$ and $f_*$). The proof is finished because
going around the diagram on the outside are the two maps
given above.
Expand Down Expand Up @@ -8831,7 +8831,7 @@ \section{Cohomology with support in a closed subset, II}
(follows by checking what happens on stalks using that $i_*$ and $i'_*$
are exact and that $\mathcal{O}_{Z, z} = \mathcal{O}_{X, z}$
and similarly for $Z'$). Hence it suffices to construct a the top
horizonal arrow in the following diagram
horizontal arrow in the following diagram
$$
\xymatrix{
Lf^* i_* R\mathcal{H}_Z(K) \ar[rr] \ar[rd] & &
Expand Down Expand Up @@ -9634,7 +9634,7 @@ \section{Derived limits}
H^{-n - 1}(E)[n + 1] \to
K_{n + 1} \to K_n \to H^{-n - 1}(E)[n + 2]
$$
Looking at the asssociated long exact cohomology sequence the claim follows if
Looking at the associated long exact cohomology sequence the claim follows if
$$
H^{m + n}(U, H^{-n - 1}(E)),\quad
H^{m + n + 1}(U, H^{-n - 1}(E)),\quad
Expand Down Expand Up @@ -10272,7 +10272,7 @@ \section{Hom complexes}
It is a good idea to think of $\SheafHom^n$ as the
sheaf of $\mathcal{O}_X$-modules of all $\mathcal{O}_X$-linear
maps from $\mathcal{L}^\bullet$ to $\mathcal{M}^\bullet$
(viewed as graded $\mathcal{O}_X$-modules) which are homogenous
(viewed as graded $\mathcal{O}_X$-modules) which are homogeneous
of degree $n$. In this terminology, we define the differential by the rule
$$
\text{d}(f) =
Expand Down
2 changes: 1 addition & 1 deletion cotangent.tex
Original file line number Diff line number Diff line change
Expand Up @@ -1114,7 +1114,7 @@ \section{The fundamental triangle}

\medskip\noindent
Consider the category $\mathcal{C}_{C/B/A}$
wich is the {\bf opposite} of the category whose objects are
which is the {\bf opposite} of the category whose objects are
$(P \to B, Q \to C)$ where
\begin{enumerate}
\item $P$ is a polynomial algebra over $A$,
Expand Down
6 changes: 3 additions & 3 deletions curves.tex
Original file line number Diff line number Diff line change
Expand Up @@ -994,7 +994,7 @@ \section{Riemann-Roch}
capped with the cycle $[X]_1$ associated to $X$ is a natural zero
cycle on $X$ with half-integer coefficients whose degree is
$\chi(X, \mathcal{O}_X)$.
The occurence of fractions in the statement of Riemann-Roch cannot
The occurrence of fractions in the statement of Riemann-Roch cannot
be avoided.

\begin{lemma}[Riemann-Roch]
Expand Down Expand Up @@ -5880,7 +5880,7 @@ \section{Contracting rational bridges}
\begin{enumerate}
\item $X' \not = \emptyset$ where $X' \subset X$ is the scheme theoretic closure
of $X \setminus C$,
\item the scheme theoretic interesection $C \cap X'$
\item the scheme theoretic intersection $C \cap X'$
has degree $2$ over $H^0(C, \mathcal{O}_C)$, and
\item $C$ has genus zero.
\end{enumerate}
Expand All @@ -5906,7 +5906,7 @@ \section{Contracting rational bridges}
}
$$
with many good properties (all of which we will use below without
futher mention). Let $y \in Y$ be the image of $\Spec(k') \to Y$.
further mention). Let $y \in Y$ be the image of $\Spec(k') \to Y$.
Then
$$
\mathcal{O}_{Y, y}^\wedge \cong k'[[s, t]]/(st)
Expand Down
Loading

0 comments on commit 0930990

Please sign in to comment.